LSAT and Law School Admissions Forum

Get expert LSAT preparation and law school admissions advice from PowerScore Test Preparation.

Homework or Lesson help relating to our Accelerated or Live Online Accelerated Courses.
 bobbiejean
  • Posts: 6
  • Joined: May 28, 2016
|
#25755
on p. 247 of the weekend course this was bothersome:

q 11 if the statements above are true, they provide the most support for which one of the following?
the words "they provide the most support" led me to believe this was a MSS. -Why not?
 Jon Denning
PowerScore Staff
  • PowerScore Staff
  • Posts: 904
  • Joined: Apr 11, 2011
|
#25761
Hi bobbiejean,

Thanks for the question! This is a Must Be True question, which, as you suggest, is synonymous with the Most Strongly Supported idea. We simply group the two into the same categorization, because functionally/operationally, there is no difference.

Not sure what your background is in terms of LSAT terminology—different companies use different nomenclature—but going forward you'll note with us the Must Be True includes both the more strongly worded "can be logically inferred" (and similar, like #4 and #21 in this drill) as well as the slightly softer "most strongly supported" (and similar, like #11 with "provide the most support for").

Thanks!
 bobbiejean
  • Posts: 6
  • Joined: May 28, 2016
|
#25784
so the MBT and MSS are operationally the same and so I need not really be concerned? it is correct?

Get the most out of your LSAT Prep Plus subscription.

Analyze and track your performance with our Testing and Analytics Package.